WILL MARK BRAINLIEST PLEASE HELP ME :(

Answers

Answer 1

Answer:

Step-by-step explanation:

6)   Statement                      Reasons

      ∠W = ∠Q                      Given

      WX = QX                      Given

     ∠1 = ∠2                          Vertically opposite angles

    ΔPXW ≅ ΔMXQ             A S A  congruent  (Angle side Angle)

7)    ∠1 = ∠4                          Given

     FH = HF                         Reflexive property

     ∠2  = ∠3                        Given

   ΔFGH ≅ ΔHEF               A S A  congruent  (Angle side Angle)


Related Questions

In a recent survey of gun control laws, a random sample of 564 women, 313 favored stricter gun control laws. In a random sample of 588 men, 307 favored stricter gun control laws. Can it be concluded at the .05 level of significance that a lower proportion of men favor stricter gun control than women

Answers

Answer:

The p-value of the test is 0.1314 > 0.05, which means that it cannot be concluded at the .05 level of significance that a lower proportion of men favor stricter gun control than women

Step-by-step explanation:

Before solving the question, we need to understand the central limit theorem, and subtraction of normal variables.

Central Limit Theorem

The Central Limit Theorem establishes that, for a normally distributed random variable X, with mean [tex]\mu[/tex] and standard deviation [tex]\sigma[/tex], the sampling distribution of the sample means with size n can be approximated to a normal distribution with mean [tex]\mu[/tex] and standard deviation [tex]s = \frac{\sigma}{\sqrt{n}}[/tex].

For a skewed variable, the Central Limit Theorem can also be applied, as long as n is at least 30.

For a proportion p in a sample of size n, the sampling distribution of the sample proportion will be approximately normal with mean [tex]\mu = p[/tex] and standard deviation [tex]s = \sqrt{\frac{p(1-p)}{n}}[/tex]

Subtraction between normal variables:

When two normal variables are subtracted, the mean is the difference of the means, while the standard deviation is the square root of the sum of the variances.

Test if a lower proportion of men favor stricter gun control than women:

At the null hypothesis, we test if the proportion is the same, that is, the difference of the proportions if 0. So

[tex]H_0: p_M - p_W = 0[/tex]

At the alternative hypothesis, we test if it is less, that is, the subtraction of the proportions is negative. So

[tex]H_1: p_M - p_W < 0[/tex]

The test statistic is:

[tex]z = \frac{X - \mu}{s}[/tex]

In which X is the sample mean, [tex]\mu[/tex] is the value tested at the null hypothesis, and s is the standard error.

In a recent survey of gun control laws, a random sample of 564 women, 313 favored stricter gun control laws.

This means that:

[tex]p_W = \frac{313}{564} = 0.555[/tex]

[tex]s_W = \sqrt{\frac{0.555*0.445}{564}} = 0.0209[/tex]

In a random sample of 588 men, 307 favored stricter gun control laws.

This means that:

[tex]p_M = \frac{307}{588} = 0.522[/tex]

[tex]s_M = \sqrt{\frac{0.522*0.478}{588}} = 0.0206[/tex]

0 is tested at the null hypothesis:

This means that [tex]\mu = 0[/tex]

From the samples:

[tex]X = p_M - p_W = 0.522 - 0.555 = -0.033[/tex]

[tex]s = \sqrt{s_M^2+s_W^2} = \sqrt{0.0206^2+0.0209^2} = 0.029[/tex]

Value of the test statistic:

[tex]z = \frac{X - \mu}{s}[/tex]

[tex]z = \frac{-0.033 - 0}{0.029}[/tex]

[tex]z = -1.12[/tex]

P-value of the test and decision:

The p-value of the test is the probability of finding a sample proportion difference greater than 0.033, which is the p-value of z = -1.12.

Looking at the z-table, z = -1.12 has a p-value of 0.1314.

The p-value of the test is 0.1314 > 0.05, which means that it cannot be concluded at the .05 level of significance that a lower proportion of men favor stricter gun control than women

Write an equation of a function with each domain:
a. [ 2 , ∞ )
b. ( -∞ , ∞ )

Answers

Answer:

a. [ 2 , ∞ )

Step-by-step explanation:

Boxers in the nineteenth century used their bare fists. In modern boxing,

fighters wear padded gloves. How do gloves protect the brain of the boxer from

injury?​

Answers

Answer:

boxers get injured but it is good to have some security on your fist and for those who do do boxing it's nice to have some security

Candice cycles 2 kilometers during each trip to work. How many trips will Candice have to
make to cycle a total of 4 kilometers?

Answers

Answer:

2

Step-by-step explanation:

Please help image attached
A) is -4 a solution to this inequality

B) choose the correct inequality

Answers

yes : there is a closed dot on -4 which means it can be -4
C : x is greater than 40, Eric has practiced more than 40 hours
Hope this helps!

Suppose a normal distribution has a mean of 26 and a standard deviation of
4. What is the probability that a data value is between 27 and 28? Round your
answer to the nearest tenth of a percent.
A. 10.3%
B. 9.3%
C. 11.3%
D
12.13%

Answers

Answer:

D,p

Step-by-step explanation:

whattttttt ,why is there no option in D

The Probability that a data value is between 27 and 28 is 9.3%.

What is Probability?

Probability is the branch of mathematics concerning numerical descriptions of how likely an event is to occur, or how likely it is that a proposition is true. The probability of an event is a number between 0 to 1,

Here, mean of the data is 26

P(0 ≤ x - μ ≤ σ/2) = 0.195

P(0 ≤ x - μ ≤ σ/4) = 0.0987

P(σ/4 ≤ x - μ ≤ σ/2) = 0.093

Thus, the Probability that a data value is between 27 and 28 is 9.3%.

Learn more about Probability from:

https://brainly.com/question/11234923

#SPJ2

Please show work. What is the volume of the figure?

Answers

Answer:

Volume = 288 in.³

Step-by-step explanation:

Volume of the solid = volume of the cube + volume of the square pyramid

V = a³ + ⅓*a²*h

a = 6 in.

h = 12 - 6 = 6 in.

Plug in the values into the volume formula

Volume = 6³ + ⅓*6²*6

Volume = 216 + 72

Volume = 288 in.³

Pamela's age is three times Jiri's age. The sum of their ages is 28 . What is Jiri's age?

Answers

Answer:

So, if we say Jiri's age is j (we can pick any letter), then Pamela's age is 2j, right?

j + 2j = 72

j is the same as 1j, but we wouldn't call you 1James, just James.

3j = 72

To isolate j, we do the opposite operation to both sides.

3j/3 = 72/3

j/j = 1, so that cancels out. 72/3 = 24.

j = 24

Jiri is 24 years old.

Answer:

Jiri is 7 years old.

explanation:

3j + j = 28

4j = 28

4j/4 = 28/4

j = 7

this can be proven:

Pamela's age: 3×7 = 21

the sum of their ages = 28

7 + 21 = 28

graph the number that represents the situation on a number line. please help with this question i will mark you brainliest!

Answers

Answer:

OMGGGG OMGGG I NEEDTHIS QUESTION TOOOOOO

Nếu đồ thị của y=f(x) đi qua điểm (0,1), và dy/dx=xsin(x^2)/y, thì f(x)= ?

Answers

См. картинку во вложении, ответ отмечен красным цветом.

Applications and Reasoning:
4. Find the sum of: 300 + 301 + 302 + 303 + 304 + ... + 1239.

Answers

hope it helps you see the attachment for further information

Sixty percent of all children in a school do not have cavities. The probability, rounded to four decimal places, that in a random sample of 9 children selected from this school, at least 6 do not have cavities is:

Answers

Answer:

Probability[Number of 6 random sample do not have cavities] = 0.8

Step-by-step explanation:

Given:

Number of student do not have cavities = 60%

Number of random sample = 9 children

Find:

Probability[Number of 6 random sample do not have cavities]

Computation:

n = 9

p = 60% = 0.6

P(At least 6)  

Probability[Number of 6 random sample do not have cavities] = 1 - P(Less than 6)  

Probability[Number of 6 random sample do not have cavities] = 1 - P(Less than or equal to 6)

Probability[Number of 6 random sample do not have cavities] = 0.8

The probability of 9 children selected from this school, at least 6 do not have cavities is 0.80.

What is probability?

Probability means possibility. It deals with the occurrence of a random event. The value of probability can only be from 0 to 1. Its basic meaning is something is likely to happen. It is the ratio of the favorable event to the total number of events.

Sixty percent of all children in a school do not have cavities.

The probability, rounded to four decimal places, that in a random sample of 9 children selected from this school, at least 6 do not have cavities will be

n = 9

p = 0.60

Then we have

[tex]\rm P = 1 - P(less \ than \ or \ equal \ 6)\\\\P = 0.8[/tex]

More about the probability link is given below.

https://brainly.com/question/795909

Angles with sides that form two pairs of opposite rays are called

Answers

The answer would be “Vertical Angles”

Vertical Angles are two angles whose sides form two pairs of opposite rays (straight lines). Vertical angles are located across from one another in the corners of the "X" formed by the two straight lines.

Angles with sides that form two pairs of opposite rays are called vertical angles.

[tex]\large\mathfrak{{\pmb{\underline{\orange{Happy\:learning }}{\orange{!}}}}}[/tex]

Which of thw following is a result of shifting circle with!!

Answers

Answer:

I need more information to answer this

Which inequality represents all values of x for which the product below is defined ?

Answers

Answer:

x ≥6

Step-by-step explanation:

Given the product:

√(x-6)*√(x+3)

The function has to be defined if x ≥0

Hence;

√(x-6)*√(x+3)≥0

Find the product

√(x-6)(x+3)≥0

Square both sides

(x-6)(x+3)≥0

x-6≥0 and x+3≥0

x≥0+6 and x ≥0 - 3

x ≥6 and x ≥-3

Hence the required inequality is x ≥6

Answer:

B. x>/ 6

Step-by-step explanation:

A P E X

A television costs $450. During a special sale, it’s marked 1⁄3 off. What’s the sale price of the TV?

Answers

Answer:

The sale price of the TV is $ 300.

Step-by-step explanation:

Given that a television costs $ 450, but during a special sale, it’s marked 1⁄3 off, to determine what’s the sale price of the TV the following calculation must be performed:

1/3 = 0.333

1 - 0.333 = 0.666

450 x 0.666 = X

300 = X

Therefore, the sale price of the TV is $ 300.

Solve the following proportion for u. (IN THE PICTURE)

round to the nearest TENTH.​

Answers

Answer:

4.9

Step-by-step explanation:

Use the butterfly method.

13*13 = 8*u

39 = 8u

(divide by 8)

4.875 = u

4.875 is about 4.9

Answer:

4.875

Step-by-step explanation:

1. multiply 13 times 3 and then 8 times u

2. set equal to each other so you get 39=8u

3. divide both sides by 8 to isolate u

4. solve 39/8 which is 4.875

HELP NO FILES PLEASE!!!

Answers

Answer:

5.6 = AC

Step-by-step explanation:

We know that the length of BD is congruent to the length of AC, therefore we can say:

BD ≅ AC

4.8 * 1.6 = 3.2 * x

7.68 ≅ 3.2x

7.68 / 3.2 ≅ x

2.4 = x

Therefore length of AC would be:

3.2 + x = AC

3.2 + 2.4 = AC

5.6 = AC

Hope this helps!

Which graph represents the function y = 2x - 4?


Please help

Answers

The 3rd one.

-4 is the y intercept and only two have the line going across (0,-4). 2 is the slope so we go up 2 and over 1. The 3rd one is the only one who has points at (0,-4) and (1,-2) as it shows we went up 2 & to the right 1.

The radius of a sphere is increasing at a rate of 5 mm/s. How fast is the volume increasing when the diameter is 40mm? Note: The volume of a sphere of radius, r, is given by V = (4/3)pi r^3

Answers

Answer:

The volume is increasing at a rate of 25,133 mm/s.

Step-by-step explanation:

Diameter is 40mm

Radius is half the diameter, so [tex]r = \frac{40}{2} = 20[/tex]

How fast is the volume increasing when the diameter is 40mm?

We have to apply implicit differentiation, of V and r in function of t. So

[tex]V = \frac{4}{3} \pi r^3[/tex]

[tex]\frac{dV}{dt} = 4\pi r^2 \frac{dr}{dt}[/tex]

The radius of a sphere is increasing at a rate of 5 mm/s.

This means that [tex]\frac{dr}{dt} = 5[/tex]

Then

[tex]\frac{dV}{dt} = 4\pi r^2 \frac{dr}{dt}[/tex]

[tex]\frac{dV}{dt} = 4\pi (20)^2(5)[/tex]

[tex]\frac{dV}{dt} = 25133[/tex]

The volume is increasing at a rate of 25,133 mm/s.

Round the following as specified. ONLY ANSWER IF YOU KNOW THE ANSWER

Answers

Answer:

153.3852

Step-by-step explanation:

one tens hundredths thousandths

so to round to the nearest thousandth you would look at the 9 to round the 1 to 2

Answer:

153.385

Step-by-step explanation:

Thousandths is the third place to the right of the decimal point.

There are no ones in decimals.

Help !!!!!!!!!!!!!!!!!!!!!!!

Answers

Answer:

<ADB = 75 degrees

Step-by-step explanation:

We know, a straight line has an angle of 180 degrees.

A = (180-143) = 37 degrees.

A + C = (37 + 38) = 75 degrees.

The angles of a triangle will always add up to 180 degrees.

<ADC = (180-75) = 105 degrees.

So,

<ADB = (180-105) = 75 degrees.

Hope, this will help. Please mark me brainliest.

Thank you.

Damian works after school. Each
day he earns a set amount, plus
an hourly wage, as shown in the
table Write a linear function i
that Damian can use to determine
his pay.

Answers

Answer:

f(x) = 12x = 10

Step-by-step explanation:

We need a linear equation in the slope-intercept form.

y = mx + b

where y = total pay, m = hourly salary, x = number of hours worked, and b = y-intercept, or initial value

Let's look in the table.

1 hour: $22

2 hours: $34

The difference in pay between 1 hour and 2 hours is $34 - $22 = $12.

The difference in time between 1 hour and 2 hours is 1 hour.

In 1 hour he earns $12. That means the slope is 12.

We know he earns $22 for working a total of 1 hour.

Start at 1 hour and $22 on the table.

Subtract 1 hour from 1 hour to get 0 hours.

Subtract $12 form $22 to get $10.

That means for 0 hours he gets $10. b = 10

The equation is

y = 12x + 10

In function form, we have:

f(x) = 12x = 10

Here we want to find a linear relation with only using the data in a table, we will find that the line is:

[tex]y = 12\cdot x + 12[/tex]

We know that Damian's earns a set amount plus an hourly wage, then this can be modeled with a linear equation:

[tex]y = a\cdot x + b[/tex]

Where a is the slope, which in this case is the hourly wage, and b is the y-intercept, which in this case is the set amount.

Such that x is the number of hours and y is the pay.

We know that if the line passes through two points (x₁, y₁) and (x₂, y₂) the slope can be given as:

[tex]a = \frac{x_2 - x_1}{y_2 - y_1}[/tex]

By looking at the table we can find two points of our line, for example, if we use the first and third points:

(1, 22) and (2, 34) then the slope will be:

[tex]a = \frac{34 - 22}{2 - 1} = 12[/tex]

Then the line is something like:

[tex]y = 12\cdot x + b[/tex]

To find the value of b, we can use one of the points, for example, the point (1, 22) means that when x = 1, we must have y = 22.

Replacing that in the above equation we have:

[tex]22 = 12\cdot1 + b\\\\22 = 12 + b\\\\22 - 12 = b = 12[/tex]

Then the equation of the line is:

[tex]y = 12\cdot x + 12[/tex]

If you want to learn more, you can read:

https://brainly.com/question/22901085

Austin picks 14 apples from the tree in his backyard. He gives his grandmother 6 apples to make a pie. Which number sentence shows how many apples Austin has left

Answers

Answer: Since he had fourteen, and he gave away six of them, he has eight left.

Step-by-step explanation:

The number sentence shows the number of apples Austin has left is 14 - 6 and equal to 8.

What is a number system?

A way to express or represent numbers is through the number system.

The Number System includes any of the several sets of symbols and the rules for using them to represent numbers.

As per the given,

Total number of apples = 14

Number of apples that make a pie = 6

Left = 14 - 6 = 8

Hence "The number sentence shows the number of apples Austin has left is 14 - 6 and equal to 8".

For more about the number system,

https://brainly.com/question/22046046

#SPJ2

Think of 5 positive integers that have a mode of 4, a median of 8, a mean of 9 and a range of 12 what are the 5 integers

Answers

Answer:

{4, 4, 8, 13, 16}

Step-by-step explanation:

If we have a set of N numbers in order:

{x₁, x₂, ..., xₙ}

The median is the middle number of the set.

The mode is the value that is repeated more times in the set

The mean is computed as:

[tex]M = \frac{x_1 + x_2 + ... + x_n}{N}[/tex]

And the range is the difference between the largest value and the smallest value.

Here we want 5 positive integers, so we have something like:

{x₁, x₂, x₃, x₄, x₅}

We know that the median is 8.

Then the middle value, x₃, is equal to 8.

We also know that the mode is 4, then we can have the number 4 repeated two times, and because 4 is smaller than 8 and this must be in order, then we can have:

x₁ = x₂ = 4

Replacing these in the set we have:

{4, 4, 8, x₄, x₅}

Now we know that the range is 12, and this was the difference between the largest and smallest value.

We know that the smallest value is 4, and the largest value is x₅, then we will have:

x₅ - 4 = 12

x₅ = 12 + 4 = 16

Then our set is:

{4, 4, 8, x₄, 16}

Finally, we know that the mean is 9.

Remember that the equation for the mean is:

[tex]M = \frac{4 + 4 + 8 + x_4 + 16}{5}[/tex]

Then we need to solve:

[tex]9 = \frac{4 + 4 + 8 + x_4 + 16}{5}[/tex]

[tex]9*5 = 4 + 4 + 8 + x_4 + 16}[/tex]

[tex]45 = 32 + x_4[/tex]

[tex]45 - 32 = x_4 = 13[/tex]

Then the set of five positive integers is:

{4, 4, 8, 13, 16}

In a bag of marbles, 1/6 are green, 1/12 are yellow, 1/2 are white and 1/4 are blue. If someone
takes a marble from the bag without looking, which color is it most likely to be?
a. White
b. Blue

Answers

Absolutely blue have more chance to get because blue have more number of marbal than the white one ok

Which point is a good approximation of a turning point
of the graph?
0 (-1.5,3)
0 (-0.5, -1)
0 (0,0)
O (1.0)

Answers

Answer:

(-0.5, -1)

Step-by-step explanation:

A turning point is a point of the graph where the graph changes from increasing to decreasing (rising to falling) or decreasing to increasing (falling to rising)

20.48=8v
Solve for v

Answers

Answer:

2.56

Step-by-step explanation:

Divide 20.48/8

I hope this helped!

Your answer to this question is, 2.56

Which best describes the orthocenter of a triangle?
• A. The point where the three altitudes of the triangle intersect
• B. The point where the three angle bisectors of the triangle intersect
O C. The point where the three medians of the triangle intersect
O D. The point where the perpendicular bisectors of the three sides of
the triangle intersect

Answers

B is the correct answer

Will give brainlest!!! Please view picture for question.

Answers

Step-by-step explanation:   6 4 12 00

V = a²(h/3)    where a is the length of a side s    and  since h = (2/3)s

V = s²(2s/3)/3

V = s³(2/9)    I'll go the D

Other Questions
6. Find the value of y. 160 60 50 Reptiles, amphibians, and fish are all cold-blooded animals. Their body temperatures are determined by their surroundings. How does a cold-blooded animals ability to digest food change in colder temperatures? Let a and b be real numbers where a 4640. Which of the following functions could represent the graph below?Of(x) = x(x a)2(x 634f(x) = x(x a) (x - b)2f(x) = AX - a)(x b)2F(X) = x2(x - a) 5(x b) help me on this question Heres another photo illPost the word problem ina minute In 1940, Mussolini ended Italys neutrality by declaring war onthe US.Germany.the USSR and the US.Great Britain and France. 5x+7=4 What is the first step in solving this equation Can someone help me please I need the mode, median, range and the mean.? Should humans colonize mars? Explain why? Si se sabe que el bronce se forma con 60% de cobre y 40% de estaocual sera la masa de cobre contenida en 200 gramos de bronce You have been out of the country studying for the schoolarship for a few months and you have just returned .Write a letter to your former headteacher to whom you are indebted because he or she facilitated award of the schoolarship if the coordinates are ( -1 , 5) and moved 9 units to the left what is the final answer? The hydrogen between the two strand of DNA are broken by the enzyme DNA helicase What information can a scientist learn directly from a single fossil? O A. What the organism looked like B. How the organism is related to organisms on Earth today C. How many offspring the organism produced D. How long the organism lived In which of the following does the land being moved contain approximately 60% water? landslide slump mudflow creep does depending on other countries help for development Time1 point2You scored the following grades on your first three math tests: 71.78, 81. You only have one test remaining. What is the highest possible test averageyou can get? ???????????????????????????????????/// please help! Find the area of the circle.Use 3.14 for pi Michael has 24 CDs. This is four more than five times the amount that Kathleen has. How many CDs does Kathleen have?